Login

Welcome, Guest. Please login or register.

April 20, 2024, 06:37:14 am

Author Topic: VCE Physics Question Thread!  (Read 607373 times)  Share 

0 Members and 2 Guests are viewing this topic.

Stevensmay

  • Guest
Re: VCE Physics Question Thread!
« Reply #225 on: October 12, 2013, 05:59:22 pm »
0
In order to replace the fuse as safely as possible, which of the following is the best precaution for Joan to
take?
A. stand on a rubber mat
B. switch off the mains supply
C. disconnect the transformer from the mains supply
D. remove the load from the transformer

Why C instead or B?
This is just my thought process.
Switch could fail and not actually disconnect power to the fuse.
Unplugging it guarantees there will be no power to the fuse, therefore no chance of electrocution.

I think this would be a case of, one answer is slightly better than the other.

Robert123

  • Victorian
  • Forum Obsessive
  • ***
  • Posts: 201
  • Respect: +5
  • School: Kyabram P-12 College
Re: VCE Physics Question Thread!
« Reply #226 on: October 12, 2013, 06:36:01 pm »
0
Photonics (detailed study) question,
Could someone please explain the what "numerical aperture" and "acceptance angle". I can usually get MC questions related to them right by applying the formula but I have no genuine idea what the numbers even mean.
Cheers
Bumping this

Energy level diagram question:
E(eV)
0         N= infinite
-1.38   N=5
X         N=4
-1.94   N=3
Y         N=2
-5.13   N=1
A photon of 3.61eV is absorb by the ion. Moments later, photons with energy 0.42eV and 1.5eV are ejected from the atom. Use the information provide to calculate the energy levels at n=2 and n=4.

For n=1 to n=4,
5.13-3.61= 1.52eV
This agree with the energy level for n=3 since
N=4 to n=3
1.94-1.52= 0.42eV
The issue I'm having is getting the right n=2 since the 1.5eV could come from n=3 (giving a value of -3.44eV) or from n=4 (giving a value of -3.02eV).  The answer gives -3.02eV but how is this more right than -3.44eV since both could be ejected.
So which one is 'more right'? If vcaa ask a question like this would they give both answers as correct or just one?

SocialRhubarb

  • Victorian
  • Forum Obsessive
  • ***
  • Posts: 313
  • Respect: +34
  • School Grad Year: 2013
Re: VCE Physics Question Thread!
« Reply #227 on: October 12, 2013, 11:23:38 pm »
+1
The key here is that you're dealing with a single atom absorbing a single photon.

If the jump from N=4 to N=2 gives us a 1.5 eV photon, it can no longer produce a photon with an energy of 0.42 eV, hence it cannot jump from N=4 to N=2 to produce a 1.5 eV photon.
Fight me.

Jaswinder

  • Victorian
  • Trendsetter
  • **
  • Posts: 152
  • Respect: 0
  • School Grad Year: 2014
Re: VCE Physics Question Thread!
« Reply #228 on: October 13, 2013, 08:26:20 am »
0
what effect does changing the slit width have on the diffraction pattern?

Stevensmay

  • Guest
Re: VCE Physics Question Thread!
« Reply #229 on: October 13, 2013, 11:51:53 am »
0
Our equation for the extent of diffraction is where w is the width of the slit.
We can see that as we increase w, becomes smaller, thus less diffraction occurs.
As we decrease w, more diffraction occurs as is now bigger.

This also explains why we need to use very thin slits/apertures for this experiment, as if they are too big any diffraction is not evident.

lolipopper

  • Victorian
  • Forum Obsessive
  • ***
  • Posts: 317
  • I'm making aaaalll kaaindzzz of gaains
  • Respect: -4
  • School: Lalor North Secondary College
  • School Grad Year: 2013
Re: VCE Physics Question Thread!
« Reply #230 on: October 13, 2013, 12:16:14 pm »
0
Our equation for the extent of diffraction is where w is the width of the slit.
We can see that as we increase w, becomes smaller, thus less diffraction occurs.
As we decrease w, more diffraction occurs as is now bigger.

This also explains why we need to use very thin slits/apertures for this experiment, as if they are too big any diffraction is not evident.

what about the intensity of the pattern? and what effect does it have on the fringe spacing?
2014: Monash University, Law

BasicAcid

  • Victorian
  • Forum Obsessive
  • ***
  • Posts: 207
  • Respect: +135
Re: VCE Physics Question Thread!
« Reply #231 on: October 13, 2013, 03:46:58 pm »
0
what about the intensity of the pattern? and what effect does it have on the fringe spacing?

No effect on the intensity and as the diffraction is larger, the diffraction patterns are also larger, i.e. the fringe spacing is larger

barydos

  • Victorian
  • Forum Obsessive
  • ***
  • Posts: 314
  • Respect: 0
  • School: Nossal High School
  • School Grad Year: 2013
Re: VCE Physics Question Thread!
« Reply #232 on: October 15, 2013, 09:25:20 pm »
0
Hello
This question came from exam 1 of the VCAA 2006 paper:
Spoiler
The assessment report's comment:
Spoiler

I'd just like to know what your responses might be, because I feel the provided comment is insufficient.
2012: Methods [47] | Chinese SL [35]
2013: Specialist [48] | Chemistry [41] | Physics [44] | English Language [40]
ATAR: 99.55

Alwin

  • Victorian
  • Forum Leader
  • ****
  • Posts: 838
  • Respect: +241
Re: VCE Physics Question Thread!
« Reply #233 on: October 15, 2013, 09:42:23 pm »
+1
offtopic
Dont be smartass alright. It was a genuine question.

i just got the physics marking scheme from my teacher. it says "where questions dont specify that the working must be shown, if the correct answer is in the box the student receives full marks". Maybe you should get some suffice information yourself before absolutely classifying other peoples question invalid and, from the sound of your tone, stupid.
Sorry lolipopper if I made you feel like you'd asked a "stupid question" or anything, I didn't mean it in that way. I was just joking with you, about the "read the instructions first" part I put in the picture. I haven't been on this thread in a while, been rather stressed for Indo oral exam so again I apologise for being or seeming a bit short with you.
I was just showing the source of my "information". I can only talk about the pre-2012 exam marking scheme, and as Tim...blahhh has confirmed from more accurate sources Re: Physics [3/4] Question Thread! it turns out I was wrong. Not going to blame the practise exam, it's just that it turns out I was wrong and I'm sorry if my response was inconsiderate.

Good luck to everyone with physics :)
2012:  Methods [48] Physics [49]
2013:  English [40] (oops) Chemistry [46] Spesh [42] Indo SL [34] Uni Maths: Melb UMEP [4.5] Monash MUEP [just for a bit of fun]
2014:  BAeroEng/BComm

A pessimist says a glass is half empty, an optimist says a glass is half full.
An engineer says the glass has a safety factor of 2.0

sin0001

  • Victorian
  • Forum Obsessive
  • ***
  • Posts: 487
  • Respect: +1
  • School Grad Year: 2013
Re: VCE Physics Question Thread!
« Reply #234 on: October 15, 2013, 10:17:45 pm »
0
With questions talking about the extent of diffraction: do we assume that max. diffraction occurs when wavelength/width ratio ~ 1 or when this ratio is a large value?
ATAR: 99.00
Monash Commerce Scholars

lolipopper

  • Victorian
  • Forum Obsessive
  • ***
  • Posts: 317
  • I'm making aaaalll kaaindzzz of gaains
  • Respect: -4
  • School: Lalor North Secondary College
  • School Grad Year: 2013
Re: VCE Physics Question Thread!
« Reply #235 on: October 15, 2013, 11:10:43 pm »
+1
offtopic
Sorry lolipopper if I made you feel like you'd asked a "stupid question" or anything, I didn't mean it in that way. I was just joking with you, about the "read the instructions first" part I put in the picture. I haven't been on this thread in a while, been rather stressed for Indo oral exam so again I apologise for being or seeming a bit short with you.
I was just showing the source of my "information". I can only talk about the pre-2012 exam marking scheme, and as Tim...blahhh has confirmed from more accurate sources Re: Physics [3/4] Question Thread! it turns out I was wrong. Not going to blame the practise exam, it's just that it turns out I was wrong and I'm sorry if my response was inconsiderate.

Good luck to everyone with physics :)

im sorry too bro. all good. and good luck to you too.

With questions talking about the extent of diffraction: do we assume that max. diffraction occurs when wavelength/width ratio ~ 1 or when this ratio is a large value?

yes, the size of the aperture must be approximately the same as that of the wavelength for significant diffraction to occur.

Mod Edit - Merged double posts (Phy124)
« Last Edit: October 16, 2013, 12:45:16 am by Phy124 »
2014: Monash University, Law

joey7

  • Victorian
  • Forum Regular
  • **
  • Posts: 74
  • Respect: 0
  • School: Notre Dame College Shepparton
  • School Grad Year: 2013
Re: VCE Physics Question Thread!
« Reply #236 on: October 16, 2013, 08:46:01 pm »
0
Quick question
"The shape of racing cars is designed to minimise air resistance. To a reasonable approximation the force (F) or air resistance is proportional to the square of the speed (v). That is: F=kv^2 where k is the drag coefficient.:

Which if the following choice is a possible unit for K
A. kgm^-1
B. s^2m^2
C. N
D. Nm^2s^-1
E. S^2M^-2

SocialRhubarb

  • Victorian
  • Forum Obsessive
  • ***
  • Posts: 313
  • Respect: +34
  • School Grad Year: 2013
Re: VCE Physics Question Thread!
« Reply #237 on: October 16, 2013, 08:51:46 pm »
+2
Force is equal to mass times acceleration, so it has the units of mass, kg, times acceleration, ms-2, so force has the units of kg m s-2.

v2 has the units of m2 s-2.

k=F/v2=kg m s-2 m-2 s2=kg m-1

So the answer is A, kg m-1.
Fight me.

lolipopper

  • Victorian
  • Forum Obsessive
  • ***
  • Posts: 317
  • I'm making aaaalll kaaindzzz of gaains
  • Respect: -4
  • School: Lalor North Secondary College
  • School Grad Year: 2013
Re: VCE Physics Question Thread!
« Reply #238 on: October 17, 2013, 08:35:25 pm »
+1
what is a good answer to the following:
-what is modulation and demodulation?
2014: Monash University, Law

Jaswinder

  • Victorian
  • Trendsetter
  • **
  • Posts: 152
  • Respect: 0
  • School Grad Year: 2014
Re: VCE Physics Question Thread!
« Reply #239 on: October 17, 2013, 09:53:53 pm »
0
I understand Question 9 and question 14 in motion  :-\
http://www.vcaa.vic.edu.au/Documents/exams/physics/physics1w06.pdf

thanks